the angle of elevation of the top of a tower from a point 42 metres away from it's base on level ground is 26 degrees. find the height of the tower.

Answers

Answer 1

Answer:

20.485 Meters

Step-by-step explanation:

So first you wanna draw a diagram.  Start with the tower, then on the ground to the left (or right) draw a point.  The point will be labeled as 42 m away from the tower.  Now draw a line from that point to the top of the tower.  This makes your triangle, and that angle you just drew that touches the point is 26 degrees.

Now, since you have a right triangle you can use trig.  You know an angle and a side.  Specifically, relative to the 26 degree angle you know the adjacent angle and want the opposite, which is the tower.  So opposite and adjacent is tangent.  So you set up tan(26) = o/42 where o is the opposite side.  

So solving you get o = 20.485 meters


Related Questions

A triangle has the vertices of P(2,5),Q(-4,7) and R(-2,-1) classify the triangle is scalene, Isoceles or an equilateral triangle

Answers

Answer:

isoceles

Step-by-step explanation:

The vertices are not equal

What is the answer to this question

Answers

26 m rhiendjfnrbsjen

Answer:

26 m is the answer to the question

Rewrite the equation 8x + 6x2 - 7= in standard form and identify a, b, and c.
Select one:
a. a =5,b= 8.c = -7
b.a=6,6= 8,0 = 1
ca=5,b=8,0= 7
da= 8,b= 6.c = -7
40

Answers

Answer:

a = 6

b = 8

c = -7

Step-by-step explanation:

In standard form, we have ;

y = ax^2 + bx + c

Here, we have;

6x^2 + 8x - 7

a is the coefficient of x^2 which is 6 in this case

b is the coefficient of x which is 8

c is the last number which is -7

So we have;

a = 6

b = 8

c = -7

GIVING 15 POINTS PLS HELP!!!

Part A: factor 2x^2b^2 + 5xb^2 + 2b^2. Show your work.
Part B: factor x^2 - 8x + 16. Show your work.
Part C: factor x^2 - 49. Show your work.

Answers

Answer:

the picture I've given is an answer

What is the simplified value of the expression below?
1/3 divided by 2/3

0

1/3

1/2

1

Answers

1/2
Is the answer
To the problem

Answer:

option C : 1/2

Step-by-step explanation:

[tex]\frac{1}{3} \div \frac{2}{3} \\\\\frac{\frac{1}{3}}{\frac{2}{3}}\\\\\frac{1}{3} \times \frac{3}{2} \\\\\frac{1}{2}[/tex]

Someone help me please I’m exhausted & struggling

Answers

Answer:

Step-by-step explanation:

Of the four choices given, which two, when written as a system, have a solution of (-4,5)?
х
-1
2
3
5
y
2
-1
-2
-4
2x+y=-3
-2x+y=-3
Х
-1
2.
3
7
0
-3
4
-8
2x+y=-3 and
Х
--1
2
3
5
y
2.
-1
-2
-4
0-2x+y=-3 and
х
-1
2
3
5
у
2.
-1
-2
-4

Answers

Answer:

both choices with 2x+y = -3

Step-by-step explanation:

to have the solution (-4, 5), that point must be on both equations/functions, meaning it must be on either one.

in other words, if the point is not on at least one of the functions, it cannot be a solution for that system.

the given function

2x + y = -3

looks like for the point (-4, 5)

2×-4 + 5 = -3

-8 + 5 = -3

-3 = -3

correct.

but

-2x + y = -3

looks like for (-4, 5)

-2×-4 + 5 = -3

8 + 5 = -3

13 = -3

wrong. the point is not on this function.

as we can therefore rule out 2 of the answer options, the other 2 most be correct.

The two equations which when written as a system has a solution of (-4, 5) is; 2x + y = -3 and 2x + y = -3

Inequalities

The correct equations must have same output with the given one when we place -4 and 5 for x and y respectively.

Now, for 2x + y = -3

At x = -4, and y = 5 we have;

2(-4) + 5 = -3

Same with the right hand side.

For -2x + y = -3;

At x = -4, and y = 5 we have;

-2(-4) + 5 = 13

Not the same with the right hand side.

Thus, the two equations with 2x + y = -3 are correct

Read more about Inequalities at; https://brainly.com/question/24372553

How many marbles do you need to be able to arrange them into the shape of an equilateral triangle with 75 rows

Answers

Answer:

2850

Step-by-step explanation:

1+2+3+4+5+6+7...74+75

just add up all the numbers from 1 to 75

What is the slope of the line? What is the y-intercept of the line? y = 2x + 5

Answers

Slope intercept form of a line is, y = mx + c where m is the slope and c is constant.

Judging the given equation y = 2x + 5

Slope (m) of the line is 2,

y-intercept of the line,

y = 2x + 5

y = 2×0 + 5

y = 5

Answered by GAUTHMATH

Answer:

m = 2

y intercept = 5

Step-by-step explanation:

The given equation of the line is ,

[tex]\implies y = 2x +5[/tex]

We know that the Standard equation of Slope Intercept Form of the line is,

[tex]\implies y = mx + c[/tex]

Where ,

m is slope c is y intercept

On comparing to the Standard form of the line we get ,

[tex]\implies Slope = 2 [/tex]

[tex]\implies y - intercept= 5[/tex]

What is the y-intercept of the line y = -3x + 7?
O A. -7
O B. 3
O C. 3
O D. 7

Answers

Answer:

Answer would be

D: 7

It’s D which is positive 7

Tariq wants to make some purple paint.
He mixes blue, red and white paint in the ratio 5:3:2
Tariq needs 60 litres of purple paint.
He gets a discounted price of 60% on the red paint.
Calculate how many litres of red paint he buys at the discounted price.

Answers

Answer:

18 liters

Step-by-step explanation:

Blue : red : White = 5 : 3 : 2

Quantity of Blue paint = 5x

Quantity of Red paint = 3x

Quantity of White paint = 2x

Total paint = 5x + 3x + 2x = 10x

10x = 60 liters

x = 60/10

x = 6

Quantity of red paint = 3x = 3*6 = 18

A corporate team-building event cost $4, plus an additional $3 per attendee. If there are 39 attendees, how much will the corporate team-building cost?

Answers

Answer:

$121

Step-by-step explanation:

Find how much additional money it will cost from the attendees:

39(3)

= 117

Add the other $4:

117 + 4

= 121

So, it will cost $121


Find angle m A. 69
B. 23
C. 46
D. 167

Answers

Answer:

a

Step-by-step explanation:

∠DAF - ∠CFA=∠DFC

 118-49 =69

If f(x) = 2x2 + 1, what is f(x) when x = 3?

Answers

Answer:

18

Step-by-step explanation:

f(x)=2x(2+1)

f(3)=2(3)(2+1)

    =6(2+1)

    =12+6

    =18

On Monday, a localamburger shop said a combined total of 225 hamburgers and cheeseburgers. The number of cheeseburgers sold was two times the number
of hamburgers sold. How many hamburgers were sold on Monday?
hamburgers
х
?

Answers

Answer:

h = hamburgers sold

2h - 72 = 578

2h = 650

2       2

h = 325

There were 72 fewer cheeseburgers sold than hamburgers. That means cheeseburgers = 325 - 72

325 - 72 =  253

253 = cheeseburgers

325 = hamburgers

253 + 325 = 578

Hamburgers = 325

Hope this helps :)

If 4/3 . sin42⁰ = x then 4/3 . cos48⁰ = ?​

Answers

Step-by-step explanation:

If 4/3 . sin42⁰ = x then 4/3 . cos48⁰ = ?

so it's that

which of the following functions is graphed below?

Answers

Answer:

d number on is the wright answer

hope it may help you

Find y

Help me please

Answers

Answer:

y = 46

Step-by-step explanation:

Y=46 that’s the answer

Each of the digits 3, 4, 6, 7, 8, and 9 is used once and only once to form a six-digit integer. What is the largest six-digit multiple of 4 that can be formed

Answers

Answer:

987436

Step-by-step explanation:

The number is divisible by 4 if the number formed by the last two digits is divisible by 4.

Since we are looking for the largest possible option, lets try 2-digit numbers formed by the given digits.

The smallest 2-digit number that is divisible by 4 is 36

Then the largest number is:

987436

maths questions on coordinate geomery

Answers

Answer:

:

Example Question #1:

Step-by-step explanation:

Which of the following is the equation of a line that is parallel to the line 4x – y = 22 and passes through the origin?

Possible Answers:

4x – y = 0

(1/4)x + y = 0

4x + 8y = 0

4x = 8y

y – 4x = 22

Correct answer:

4x – y = 0

Explanation:

We start by rearranging the equation into the form y = mx + b (where m is the slope and b is the y intercept); y = 4x – 22

Now we know the slope is 4 and so the equation we are looking for must have the m = 4 because the lines are parallel. We are also told that the equation must pass through the origin; this means that b = 0.

In 4x – y = 0 we can rearrange to get y = 4x. This fulfills both requirements.

', .

The length of a rectangle is 7 ft less than three times the width, and the area of the rectangle is . Find the dimensions of the rectangle.

Answers

Answer:

See explanation

Step-by-step explanation:

the length of a rectangle is 7 feet less than three times it’s width. if the area of the rectangle is 180 ft, find the dimensions of the rectangle.

Width = x

Length = 3x - 7

Area of a rectangle = length × width

180 = (3x - 7) * x

180 = 3x² - 7x

3x² - 7x - 180 = 0

Using quadratic formula

a = 3

b = -7

c = -180

x = -b ± √b² - 4ac / 2a

x = -(-7) ± √(-7)² - 4(3)(-180) / 2(3)

x = 7 ± √49 - (-2160) / 6

= 7 ± √49 + 2160 / 6

x = 7 ± √2209/6

= 7 ± 47 / 6

= (7 + 47)/6 or ( 7 - 47)/6

= 54/6 or -40/6

x = 9 or -6.67

x cannot be negative

Therefore,

x = 9 ft

Width = x = 9 ft

Length = 3x - 7

=3(9) - 7

= 27 - 7

= 20 ft

Find the constant of proportionality if y is
proportional to x.

A. 25
B. 21
C. 26
D. 28

HELPPP!!!!!!

Answers

B. 21 because 21+21=42 42+21=63 etc

I need help with this, Im never good with these

Answers

Answer:

(a) basketball

(b) hat

(c) stuffed animal

explanation:

I'm not sure if got right but from what i see;

Chances from 1-23 you get a stuffed boi

Chances from 24-54 you get a hat (why)

Chances from 55-100 you get a basketball

Winner 5 got 90, which is between 55-100

Winner 6 got 35, which is between 24-54

winner 8 got a 7, which is between 1-23

Hope I didn't read it wrong

Which expression is equivalent to −10x−10+2x+9?

Answers

Answer:

-8x - 1

General Formulas and Concepts:

Algebra I

Terms/Coefficients

Step-by-step explanation:

Step 1: Define

Identify

-10x - 10 + 2x + 9

Step 2: Simplify

Combine like terms (x):                                                                                        -8x - 10 + 9Combine like term:                                                                                              -8x - 1

[tex]\huge\textsf{Hey there!}[/tex]

[tex]\large\textsf{-10x - 10 + 2x + 9}[/tex]

[tex]\huge\textsf{COMBINE the LIKE TERMS}[/tex]

[tex]\large\textsf{-10x + 2x - 10 + 9}[/tex]

[tex]\large\textsf{-10x + 2x}\\\\\large\textsf{ = \bf -8x}[/tex]

[tex]\large\textsf{-10 + 9}\\\\\large\textsf{ = \bf -1}[/tex]

[tex]\boxed{= \large\textsf{\bf -8x - 1}}\large\checkmark[/tex]

[tex]\boxed{\boxed{\huge\textsf{Answer: \bf -8x - 1 }}}\huge\checkmark[/tex]

[tex]\large\textsf{Good luck on your assignment and enjoy your day!}[/tex]

~[tex]\frak{Amphitrite1040:)}[/tex]

Find the greatest number such that; x360y is divisible by 24
WILL GIVE BRAINIEST, NEED ANSWER ASAP

Answers

Answer:

93600

Step-by-step explanation:

24=3*2*2*2

y=0,8

1) x3600, then x=9,6,3

2)x3608, then x=1,4,7

9 is the largest so the final answer is 93600

3 years later a mother will be 4 times as
old as her son. 3 years ago ,the mother age was two times as old as her son age will be 8 years hence. What are their present ages?​

Answers

Answer:

33 and 6 years

Step-by-step explanation:

Mother's age = mSon's age = s

3 years later:

m + 3 = 4(s + 3) ⇒ m = 4s + 12 - 3 = 4s + 9

3 years ago:

m - 3 = 2(s + 8) ⇒ m = 2s + 18 + 3 = 2s + 21

Solve for s by substitution:

4s + 9 = 2s + 212s = 12s = 6

Find the value of m:

m = 4*6 + 9 = 33

Mother is 33 and son is 6

In the equation 17x2 = 12x, the value of c is:
O
0 12
O 17

Answers

Answer:

ok ok ok ok ok ok ok

Step-by-step explanation:

In a group of 26 pupils, 3 play the flute only.
5 play the piano only.
7 play neither instrument.
How many pupils play both instruments?

Answers

26 minus 3 is 23 , 23 minus 5 is 18, 18 minus 7 is 11 so i’m pretty sure


answer =11

Here is a Venn diagram:

Regan earns $1380 per month out of which he saves $1140 per month. What is the ratio (in the lowest form) of Regan's income to his savings? $$

Answers

Answer:

23:19

Step-by-step explanation:

This questions is looking for this ratio:

income:savings

Regan's income is $1,380 per month.

Regan's savings is $1,140 per month.

Substitue those numbers in our original ratio, and you'll get this:

1380:1140

However, this question wants the lowest form of the ratio.

To do that, you can find the GCF (greatest common factor) and divide both number's by it, or you can just randomly divide them by small numbers you know both of them are divisible by.

I'm going to use the 2nd method because it's usually quicker for big numbers like these:

1380/10 = 138

1140/10 = 114

138/2 = 69

114/2 = 57

69/3 = 29

57/3 = 19

29 and 19 are prime numbers, so you can't divide them anymore. This is how we know we've reached the lowest terms of these numbers.

So the answer is 29:19

Hope this helps (●'◡'●)

Answer: 23:19

Step-by-step explanation: 1380 and 1140 LCM is 60. 1380/6 = 23 and 1140/6= 19

Therefore, the answer is 23:19

what is the value of x? (3x-14)°=180° [4(x-9)]°=180°

Answers

Answer:

3x-14=180

3x=194

x= 64 2/3

4(x-9)=180

4x-36=180

4x=216

x=54

Hope This Helps!!!

Step-by-step explanation:

(3x-14)°=180°

3x-14=180

3x=180+14

3x=194

x=64.6

[4(x-9)]°=180°

4x-36=180

4x=180+36

4x=216

x=54

Other Questions
Examine life in the early factories. How were families affected by this new lifestyle? Find the scale ratio for the map described below.1cm (map) 50km (actual)The scale ratio is 1 to .....? Cmo est tu bistec?Nuestro bistec est delicioso.Mo bistec est delicioso.Tu bistec est delicioso.Mi bistec est delicioso. During the process of genetic recombination, crossing over occurs between two nonsister chromatids. DNA segments are exchanged between two nonsister chromatids during meiosis I. None of the above. maternal and paternal chromosomes pair and physically exchange DNA segments. All of the above. help me with this problem please!!!! what is the approximate value of x in the diagram below? Which item is an example of a secondary source? Suppose you invest a certain amount of money in account that earns 3% annual interest. You also invest that same amount + $2000 that earns 4% annual interest. If the total interest from both accounts at the end of the year is $535, how much has been invested in each account? Swifty Corporation purchased a truck at the beginning of 2020 for $109600. The truck is estimated to have a salvage value of $4100 and a useful life of 123000 miles. It was driven 18000 miles in 2020 and 26000 miles in 2021. What is the depreciation expense for 2020?a. $37752 b. $22308 c. $16639 d. $15444 Judy, a 28 y/o, presents to the clinic with a fever, vaginal discharge, and pain in the lower abdomen, pelvis, and lower back. These symptoms are accompanied by chills, nausea, and vomiting. This presentation is most typical of: What is theenergy2) The energy transition from n = 1 n= 3 in hydrogen is 12.09 eV (1.6022 X 10J= 1 eV)-of light emitted from n = 3 n=1?E e fiz In a right triangle, the lengths of the two legs are 8 cm and 10 cm respectively. Find the hypotenuse of the triangle.9 cm10.5 cm12 cm12.8 cm According to Okun's law, if the unemployment rate goes from 5% to 3%, what will be the effect on the GDP?A. It will increase by 7%.B. It will decrease by 7%.C. It will decrease by 1%.D. It will increase by 1%. amy shoots a 100 arrows at a target each arrow hits with a probability 0.01 what is the probability that one of her first 5 arrows hit the target Amy, a high-strung teenager, was suddenly startled by a loud bang that sounded like a gunshot. Her heartbeat accelerated rapidly. When she realized that the noise was only a car backfiring, she felt greatly relieved but her heart kept beating heavily for several minutes more. What hormone was released into her bloodstream that is causing this response While some property owners choose to perform both the property and asset manager functions themselves, many commercial property owners choose to employ professional property managers instead. The property manager works under a management contract in which the manager is empowered to serve as the owner's fiduciary. This type of relationship is more commonly referred to as a(n) Suppose a life insurance company sells a $240,000 one-year term life insurance policy to a 19-year-old female for $240. The probability that the female survives the year is 0.999578. Compute and interpret the expected value of this policy to the insurance company. The expected value is $ (Round two decimal places as needed.) Which statement best describes the effect of the alliteration in this excerpt? A 80C la presion de vapor del benceno (C6H6) es de 1 atm. Calcula la cantidad de hexano (C6H14) que debemos aadir a 200g de benceno para que su presion de vapor sea de 700mm de Hg CAN SOME HELP ME PLEASE AND THANK YOU